Difference between revisions of "2017 AMC 10B Problems/Problem 9"

(Created page with "==Problem== Placeholder ==Solution== Placeholder ==See Also== {{AMC10 box|year=2017|ab=B|num-b=8|num-a=10}} {{MAA Notice}}")
 
(Problem)
Line 1: Line 1:
 
==Problem==
 
==Problem==
Placeholder
+
A radio program has a quiz consisting of <math>3</math> multiple-choice questions, each with <math>3</math> choices. A contestant wins if he or she gets <math>2</math> or more of the questions right. The contestant answers randomly to each question. What is the probability of winning?
 +
 
 +
<math>\textbf{(A)}\ \frac{1}{27}\qquad\textbf{(B)}\ \frac{1}{9}\qquad\textbf{(C)}\ \frac{2}{9}\qquad\textbf{(D)}\ \frac{7}{27}\qquad\textbf{(E)}\ \frac{1}{2}</math>
  
 
==Solution==
 
==Solution==

Revision as of 13:36, 16 February 2017

Problem

A radio program has a quiz consisting of $3$ multiple-choice questions, each with $3$ choices. A contestant wins if he or she gets $2$ or more of the questions right. The contestant answers randomly to each question. What is the probability of winning?

$\textbf{(A)}\ \frac{1}{27}\qquad\textbf{(B)}\ \frac{1}{9}\qquad\textbf{(C)}\ \frac{2}{9}\qquad\textbf{(D)}\ \frac{7}{27}\qquad\textbf{(E)}\ \frac{1}{2}$

Solution

Placeholder

See Also

2017 AMC 10B (ProblemsAnswer KeyResources)
Preceded by
Problem 8
Followed by
Problem 10
1 2 3 4 5 6 7 8 9 10 11 12 13 14 15 16 17 18 19 20 21 22 23 24 25
All AMC 10 Problems and Solutions

The problems on this page are copyrighted by the Mathematical Association of America's American Mathematics Competitions. AMC logo.png